RegistrierenRegistrieren   LoginLogin   FAQFAQ    SuchenSuchen   
Erwartungswert des Impulses
Gehe zu Seite 1, 2  Weiter 
Neue Frage »
Antworten »
    Foren-Übersicht -> Quantenphysik
Autor Nachricht
frage1



Anmeldungsdatum: 20.02.2021
Beiträge: 569
Wohnort: bayern

Beitrag frage1 Verfasst am: 25. Jun 2022 15:24    Titel: Erwartungswert des Impulses Antworten mit Zitat

Hallo!

Es geht hier wieder um den Erwartungswert des Impulses. Ich hatte Schwierigkeiten beim Ausrechnen. Irgendetwas ist hier falsch, nur kann ich nicht erkennen, wo der Rechenfehler ist. Der Ansatz an sich stimmt ja, aber ich komme nicht auf die Lösung -hk/2.
Hat jemand eine Idee? Kann jemand einen Blick werfen?


Zuletzt bearbeitet von frage1 am 28. Jun 2022 14:17, insgesamt einmal bearbeitet
Myon



Anmeldungsdatum: 04.12.2013
Beiträge: 5873

Beitrag Myon Verfasst am: 25. Jun 2022 15:47    Titel: Antworten mit Zitat

Steht bei der erwähnten Aufgabe 51 noch etwas? Ohne zu wissen, um welche Anordnung es geht, ist für mich unklar, über welches Intervall integriert werden muss. Eine Normierung über alle reellen Zahlen ist ja nicht möglich, wenn ich mich nicht täusche.
index_razor



Anmeldungsdatum: 14.08.2014
Beiträge: 3259

Beitrag index_razor Verfasst am: 25. Jun 2022 16:26    Titel: Antworten mit Zitat

Es muß ein Intervall der der Länge sein, sonst ist nicht hermitesch und nicht reell. Mehr Information benötigt man nicht, soweit ich das sehe.

Dann bekomme ich aber einen positiven Wert . (Das ergibt für mich auch Sinn, weil die Amplitude für den positiven Wert größer ist als für den negativen.)

_________________
It is just this lack of connection to a concern with truth -- this indifference to how things really are -- that I regard as of the essence of bullshit. -- Harry G. Frankfurt
index_razor



Anmeldungsdatum: 14.08.2014
Beiträge: 3259

Beitrag index_razor Verfasst am: 25. Jun 2022 17:12    Titel: Re: Erwartungswert des Impulses Antworten mit Zitat

frage1 hat Folgendes geschrieben:
Hallo!

Es geht hier wieder um den Erwartungswert des Impulses. Ich hatte Schwierigkeiten beim Ausrechnen. Irgendetwas ist hier falsch, nur kann ich nicht erkennen, wo der Rechenfehler ist. Der Ansatz an sich stimmt ja, aber ich komme nicht auf die Lösung -hk/2.
Hat jemand eine Idee? Kann jemand einen Blick werfen?


In der letzten Zeile hast du beim Ausklammern von einen Vorzeichenfehler gemacht. Abgesehen davon sieht die Rechnung richtig aus. (Es stört allerdings ein bißchen, daß sowohl ein Intervall als auch seine Länge bezeichnet. EDIT: Oh, und warum schreibst du , anstatt ?)

Woher kommt die Lösung?

_________________
It is just this lack of connection to a concern with truth -- this indifference to how things really are -- that I regard as of the essence of bullshit. -- Harry G. Frankfurt
frage1



Anmeldungsdatum: 20.02.2021
Beiträge: 569
Wohnort: bayern

Beitrag frage1 Verfasst am: 25. Jun 2022 20:49    Titel: Antworten mit Zitat

Index_Razor, da kommt tatsächlich ein positiver Wert raus. Habe die falsche Lösung aufgeschrieben. Also es kommt +hk/2.
Aber ich versteh ehrlich gesagt nicht, wie ich auf hk/2 kommen soll.
Hast du hier die e-funktion substituiert? Oder wie hast du`s zusmmanegefasst?

Edit: mein aktueller stand sieht so aus. Hab das Vorzeichen in der letzten Zeile (im Klammer) geändert.


Zuletzt bearbeitet von frage1 am 25. Jun 2022 22:21, insgesamt einmal bearbeitet
index_razor



Anmeldungsdatum: 14.08.2014
Beiträge: 3259

Beitrag index_razor Verfasst am: 25. Jun 2022 21:24    Titel: Antworten mit Zitat

Die einfachste Möglichkeit darauf zu kommen ist folgende. und sind Eigenzustände zu verschiedenen Impulsen, nämlich und . Also müssen sie orthogonal sein



(Das steht im Zusammenhang mit der erwähnten Hermitezität von .) Nun ist aber . (Dies sind auf 1 normierte Wellenfunktionen. Die von dir angegebenen waren es nicht.) Damit ist



und



***

Du kannst es aber auch zu Fuß über das Integral in der Ortsdarstellung ausrechnen, wie du es gemacht hast. Die Rechnung ist jetzt in Ordnung, soweit ich das sehe. Du mußt nichts weiter tun als das Integral auszuführen.

_________________
It is just this lack of connection to a concern with truth -- this indifference to how things really are -- that I regard as of the essence of bullshit. -- Harry G. Frankfurt
Myon



Anmeldungsdatum: 04.12.2013
Beiträge: 5873

Beitrag Myon Verfasst am: 25. Jun 2022 21:32    Titel: Antworten mit Zitat

Die Länge des Intervalls muss aber tatsächlich gleich n*pi/k sein (was noch nirgends steht), damit der Beitrag von



gleich null wird.
frage1



Anmeldungsdatum: 20.02.2021
Beiträge: 569
Wohnort: bayern

Beitrag frage1 Verfasst am: 25. Jun 2022 21:36    Titel: Antworten mit Zitat

Das heißt, ich muss hier die substitutionsregel anwendenm und die Grenzen dementsprechend anpassen?

Edit: ich habe dazu gerade eben die komplette Lösung von einem Studienkollegen bekommen. Jetzt bin ich noch mehr verwirrt als zuvor. Was genau passiert hier.


Zuletzt bearbeitet von frage1 am 28. Jun 2022 14:17, insgesamt 2-mal bearbeitet
index_razor



Anmeldungsdatum: 14.08.2014
Beiträge: 3259

Beitrag index_razor Verfasst am: 25. Jun 2022 21:40    Titel: Antworten mit Zitat

frage1 hat Folgendes geschrieben:
Das heißt, ich muss hier die substitutionsregel anwendenm und die Grenzen dementsprechend anpassen?


Nein, du mußt nichts substituieren. Einfach das Integral ausrechnen. Ein wichtiger Hinweis steht in Myons letztem Beitrag.

P.S.: Deine Rechnung ist schon ok. Wenn dich die andere verwirrt, vergiß sie erstmal. (Sie ist auch nicht ganz einwandfrei.)

_________________
It is just this lack of connection to a concern with truth -- this indifference to how things really are -- that I regard as of the essence of bullshit. -- Harry G. Frankfurt
frage1



Anmeldungsdatum: 20.02.2021
Beiträge: 569
Wohnort: bayern

Beitrag frage1 Verfasst am: 25. Jun 2022 22:24    Titel: Antworten mit Zitat

Ich versteh´schon auf was ihr hinaus wollt. Aber muss ich dann nicht (-2 i sin(2kx) integrieren? Wenn ich für x= n pu/ k einsetze, wird der Ausdruck 0, aber ich muss zuerst diesen Ausdruck integrieren, damit ich überhaupt die Grenzen einsetzen darf?


DC1F1FEB-FDD1-4678-AB4B-DC9EDE54B321.jpeg
 Beschreibung:

Download
 Dateiname:  DC1F1FEB-FDD1-4678-AB4B-DC9EDE54B321.jpeg
 Dateigröße:  72.94 KB
 Heruntergeladen:  98 mal

index_razor



Anmeldungsdatum: 14.08.2014
Beiträge: 3259

Beitrag index_razor Verfasst am: 25. Jun 2022 22:52    Titel: Antworten mit Zitat

frage1 hat Folgendes geschrieben:
Ich versteh´schon auf was ihr hinaus wollt. Aber muss ich dann nicht (-2 i sin(2kx) integrieren?


Doch natürlich mußt du das integrieren. In deiner Rechnung hast du das Integral gar nicht ausgeführt. Ab der dritten Zeile von unten ergibt es keinen Sinn mehr.

Übrigens, woher hast du die Grenzen 0 und ? In der Aufgabenstellung, die du gepostet hast, stehen die nicht. Und wie ist definiert?

_________________
It is just this lack of connection to a concern with truth -- this indifference to how things really are -- that I regard as of the essence of bullshit. -- Harry G. Frankfurt
frage1



Anmeldungsdatum: 20.02.2021
Beiträge: 569
Wohnort: bayern

Beitrag frage1 Verfasst am: 25. Jun 2022 23:12    Titel: Antworten mit Zitat

Habt ihr nicht gesagt, dass die Länge des Intervalls gleich n*p/ k sein muss? Dann habe ich einfach np/k für die obere Grenze eingesetzt. Ich bin wirklich sehr verwirrt momentan…

Edit: wenn ich integriere, kommt folgendes heraus.



920B364A-4E9A-49A2-887B-3FE92E2F4AA1.jpeg
 Beschreibung:

Download
 Dateiname:  920B364A-4E9A-49A2-887B-3FE92E2F4AA1.jpeg
 Dateigröße:  57.68 KB
 Heruntergeladen:  84 mal

index_razor



Anmeldungsdatum: 14.08.2014
Beiträge: 3259

Beitrag index_razor Verfasst am: 26. Jun 2022 08:25    Titel: Antworten mit Zitat

frage1 hat Folgendes geschrieben:
Habt ihr nicht gesagt, dass die Länge des Intervalls gleich n*p/ k sein muss?


Ja, das ist der Abstand zwischen oberer und unterer Grenze, und das n ist unbekannt, genauso wie die untere Grenze. Steht dazu nichts in der Aufgabenstellung? Bitte poste mal die erwähnte Aufgabe 51.

_________________
It is just this lack of connection to a concern with truth -- this indifference to how things really are -- that I regard as of the essence of bullshit. -- Harry G. Frankfurt
frage1



Anmeldungsdatum: 20.02.2021
Beiträge: 569
Wohnort: bayern

Beitrag frage1 Verfasst am: 26. Jun 2022 09:01    Titel: Antworten mit Zitat

Das ist die Aufgabe 51). Ich hoffe das hilft weiter


FF688DB0-A017-4BB7-AC54-9BE31E3A37CD.jpeg
 Beschreibung:

Download
 Dateiname:  FF688DB0-A017-4BB7-AC54-9BE31E3A37CD.jpeg
 Dateigröße:  36.72 KB
 Heruntergeladen:  85 mal

index_razor



Anmeldungsdatum: 14.08.2014
Beiträge: 3259

Beitrag index_razor Verfasst am: 26. Jun 2022 09:09    Titel: Antworten mit Zitat

Steht irgendwo auf welchem Intervall die Wellenfunktionen definiert sein sollen? Steht irgendwo irgendetwas über ?
_________________
It is just this lack of connection to a concern with truth -- this indifference to how things really are -- that I regard as of the essence of bullshit. -- Harry G. Frankfurt
frage1



Anmeldungsdatum: 20.02.2021
Beiträge: 569
Wohnort: bayern

Beitrag frage1 Verfasst am: 26. Jun 2022 09:32    Titel: Antworten mit Zitat

Nein, da steht überhaupt nichts. Ich kann die Lösung von der Aufgabe eventuell noch hochladen.
index_razor



Anmeldungsdatum: 14.08.2014
Beiträge: 3259

Beitrag index_razor Verfasst am: 26. Jun 2022 09:46    Titel: Antworten mit Zitat

frage1 hat Folgendes geschrieben:
Nein, da steht überhaupt nichts.


Das ist ehrlich gesagt ein bißchen schwer zu glauben. Aber gut, ab jetzt nehmen wir an die untere Grenze liege bei irgendeinem unbestimmten , die obere Grenze sei und



mit beliebigem .

Versuche mal mit diesen Informationen das Integral auszurechnen.

Zitat:
Ich kann die Lösung von der Aufgabe eventuell noch hochladen.


Klar, schaden kann es nicht.

_________________
It is just this lack of connection to a concern with truth -- this indifference to how things really are -- that I regard as of the essence of bullshit. -- Harry G. Frankfurt
frage1



Anmeldungsdatum: 20.02.2021
Beiträge: 569
Wohnort: bayern

Beitrag frage1 Verfasst am: 26. Jun 2022 10:59    Titel: Antworten mit Zitat

Ich habe ja oben die Lösung hochgeladen. Vielleicht hilft uns die Lösung weiter. Wir können ja versuchen die Lösung zu verstehen.
Und da sind die grenzen wirklich nicht angegeben. Da steht wirklich nichts, nur das was ich hochgeladen habe.
Myon



Anmeldungsdatum: 04.12.2013
Beiträge: 5873

Beitrag Myon Verfasst am: 26. Jun 2022 11:26    Titel: Antworten mit Zitat

Falls Du die Lösung meinst, die Du vom Studienkollegen erhalten hast: ich verstehe Verschiedenes nicht:

-3. Zeile: die i als Faktoren sollten sich wegheben.

-4. Zeile: (in der Aufgabe waren es noch ) in der ersten Klammer müssten komplex konjugiert sein, und der Faktor 1/(4*Omega) müsste sich rauskürzen. Und was ist mit dem passiert?

-Weshalb wird auf der letzten Zeile plötzlich wieder integriert, und weshalb sollte das Integral ergeben?
index_razor



Anmeldungsdatum: 14.08.2014
Beiträge: 3259

Beitrag index_razor Verfasst am: 26. Jun 2022 11:33    Titel: Antworten mit Zitat

frage1 hat Folgendes geschrieben:
Ich habe ja oben die Lösung hochgeladen. Vielleicht hilft uns die Lösung weiter. Wir können ja versuchen die Lösung zu verstehen.


Welche Lösung? Mit der Information, die ich dir gegeben habe, kannst du die Aufgabe lösen. Ich bin auch davon überzeugt, daß sein muß. Ich kann mir aber nicht vorstellen, daß nirgendwo irgendeine Information zu diesem stehen soll. Woher hast du und dein Studienkollege denn die Idee, daß dieses irgendwas mit dem Integrationsgebiet zu tun haben soll? Schließlich habt ihr beide es an das Integralzeichen geschrieben.

P.S. Ich denke nicht, daß wir die Lösung vom Studienkollegen diskutieren sollten. Die ist inkorrekt und führt nur noch mehr Konfusion ein.

_________________
It is just this lack of connection to a concern with truth -- this indifference to how things really are -- that I regard as of the essence of bullshit. -- Harry G. Frankfurt
frage1



Anmeldungsdatum: 20.02.2021
Beiträge: 569
Wohnort: bayern

Beitrag frage1 Verfasst am: 26. Jun 2022 20:44    Titel: Antworten mit Zitat

Ich stimme dir da ja zu. Die Lösung gefällt mir auch nicht. Als ich die Lösung sah war ich noch mehr verwirrt. Und es kann schon sein, dass die Lösung inkorrekt ist, passiert ja nicht selten, dass die Lösungen falsch sind.
Auf jeden Fall sieht mein aktueller Stand folgendermaßen aus:
Ich bin mir sicher, dass ich wieder falsch gerechnet habe. Ich stehe auf der Leitung



23D92328-497C-4F27-A606-41CCC68B39F5.jpeg
 Beschreibung:

Download
 Dateiname:  23D92328-497C-4F27-A606-41CCC68B39F5.jpeg
 Dateigröße:  83.33 KB
 Heruntergeladen:  89 mal

Myon



Anmeldungsdatum: 04.12.2013
Beiträge: 5873

Beitrag Myon Verfasst am: 27. Jun 2022 08:50    Titel: Antworten mit Zitat

Eigentlich ist es doch nun fast richtig. Die erste Zeile würde ich nicht schreiben, zumindest nicht so. Das Skalarprodukt wird in diesem Fall ja gebildet, indem über eine Intervalllänge n*pi/k integriert wird für ein bestimmtes n.
Gehen wir von hier aus:



Nehmen wir im Folgenden an, n sei 1. Eigentlich musst Du den Term mit dem Sinus nicht integrieren, da klar ist, dass er keinen Beitrag liefert. Integriert wird ja über eine (bzw. n) Perioden, und die Fläche unter einer Sinuskurve über eine Periode hinweg ist gleich 0.
Wenn man doch substituieren möchte (nun ):






Zuletzt bearbeitet von Myon am 27. Jun 2022 09:16, insgesamt 2-mal bearbeitet
index_razor



Anmeldungsdatum: 14.08.2014
Beiträge: 3259

Beitrag index_razor Verfasst am: 27. Jun 2022 08:57    Titel: Antworten mit Zitat

Du möchtest berechnen:



Das erste Integral über stellt hoffentlich kein Problem dar. Also konzentrieren wir uns auf das zweite Integral. Was ergibt



@Myon: der eigentliche Integrationsschritt sieht in der Rechnung ziemlich konfus aus. Das fängt schon damit an, daß zwei verschiedene Bedeutungen hat.

Sicher, man kann wissen, daß dieses Integral über den Sinus ohnehin null ergibt (für alle n). Aber nur wenn man in der Lage ist, das auch auszurechnen.

_________________
It is just this lack of connection to a concern with truth -- this indifference to how things really are -- that I regard as of the essence of bullshit. -- Harry G. Frankfurt
Myon



Anmeldungsdatum: 04.12.2013
Beiträge: 5873

Beitrag Myon Verfasst am: 27. Jun 2022 09:15    Titel: Antworten mit Zitat

index_razor hat Folgendes geschrieben:
@Myon: der eigentliche Integrationsschritt sieht in der Rechnung ziemlich konfus aus. Das fängt schon damit an, daß zwei verschiedene Bedeutungen hat.

Falls Du meine Rechnung meinst, danke ja, hatte einen Vorfaktor vergessen. Und sonst, ja, die Notation mit den verschiedenen Omegas ist natürlich nicht gut. Wollte übrigens auch nicht ständig reinpfuschen und Verwirrung stiften, wollte eher Arbeit abnehmen (und muss mich manchmal etwas selbst beschäftigen)...
index_razor



Anmeldungsdatum: 14.08.2014
Beiträge: 3259

Beitrag index_razor Verfasst am: 27. Jun 2022 10:17    Titel: Antworten mit Zitat

Myon hat Folgendes geschrieben:
index_razor hat Folgendes geschrieben:
@Myon: der eigentliche Integrationsschritt sieht in der Rechnung ziemlich konfus aus. Das fängt schon damit an, daß zwei verschiedene Bedeutungen hat.

Falls Du meine Rechnung meinst, danke ja, hatte einen Vorfaktor vergessen.


Nein, deine Rechnung sah überhaupt nicht konfus aus. Ich meinte die letzte Rechnung von frage1. Da ging beim Integrieren ziemlich viel drunter und drüber und es erschien mir eben nicht "fast richtig"; z.B. war plötzlich , und die Grenzen waren auch nicht richtig eingesetzt. Deswegen dachte ich es wäre vielleicht eine gute Idee das Sinus-Integral mal Schritt für Schritt durchzurechnen.

Zitat:

Und sonst, ja, die Notation mit den verschiedenen Omegas ist natürlich nicht gut. Wollte übrigens auch nicht ständig reinpfuschen und Verwirrung stiften, wollte eher Arbeit abnehmen (und muss mich manchmal etwas selbst beschäftigen)...


Deine Hinweise sind absolut willkommen. Und frage1 scheinen sie auch zu helfen, was die Hauptsache ist. Generell finde ich es für den Fragesteller eher vorteilhaft, wenn mehr als einer antwortet.

_________________
It is just this lack of connection to a concern with truth -- this indifference to how things really are -- that I regard as of the essence of bullshit. -- Harry G. Frankfurt
frage1



Anmeldungsdatum: 20.02.2021
Beiträge: 569
Wohnort: bayern

Beitrag frage1 Verfasst am: 27. Jun 2022 13:51    Titel: Antworten mit Zitat

Ich stimme dir @index_razor vollkommen zu. Oft ist es sogar besser, wenn mehr als einer antwortet. Ich bin wirklich sehr dankbar, dass ihr mir die Aufgabe schrittweise erklärt. Nur so kommt man weiter. Also alle Hinweise sind willkommen.

Zur Aufgabe: ich versteh ehrlich gesagt nicht wie man auf Wuzel aus 3 i h k /4 pi kommt. Wenn ich 2 k integriere, dann kommt ka 2kx heraus. Und wenn ich die Grenzen einsetze, kommt pi/k heraus. Denke ich da falsch?

Edit: Ok, ich hab doch verstanden wie man auf Wuzel aus 3 i h k /4 pi kommt. Ich integriere schnell sin und melde mich


Zuletzt bearbeitet von frage1 am 27. Jun 2022 16:02, insgesamt einmal bearbeitet
frage1



Anmeldungsdatum: 20.02.2021
Beiträge: 569
Wohnort: bayern

Beitrag frage1 Verfasst am: 27. Jun 2022 15:45    Titel: Antworten mit Zitat

Wenn ich sin integriere und die Grenzen einsetze, dann kommt ja cos(2ka+2pi)-cos(2ka). Was soll ich nun für a und k einsetzen? Welche Werte?
Mir ist noch nicht klar, warum der Term mit dem Sin keinen Beitrag liefert und warum die Fläche unter einer Sinuskurve gleich 0 ist.


Zuletzt bearbeitet von frage1 am 27. Jun 2022 15:54, insgesamt 2-mal bearbeitet
index_razor



Anmeldungsdatum: 14.08.2014
Beiträge: 3259

Beitrag index_razor Verfasst am: 27. Jun 2022 15:53    Titel: Antworten mit Zitat

frage1 hat Folgendes geschrieben:
Wenn ich sin integriere und die Grenzen einsetze, dann kommt ja cos(2ka+2pi)-cos(2ka). Was soll ich nun für a und k einsetzen? Welche Werte?


Der Kosinus ist eine -periodische Funktion. Spielen die Werte von k und a also eine Rolle?

_________________
It is just this lack of connection to a concern with truth -- this indifference to how things really are -- that I regard as of the essence of bullshit. -- Harry G. Frankfurt
frage1



Anmeldungsdatum: 20.02.2021
Beiträge: 569
Wohnort: bayern

Beitrag frage1 Verfasst am: 27. Jun 2022 15:56    Titel: Antworten mit Zitat

Warum spielen die keine Rolle? Bei 2pi hat der Graph der Sin-Kurve eine Nullstelle, hat das vllt. damit zu tun?

Zuletzt bearbeitet von frage1 am 27. Jun 2022 16:05, insgesamt 2-mal bearbeitet
index_razor



Anmeldungsdatum: 14.08.2014
Beiträge: 3259

Beitrag index_razor Verfasst am: 27. Jun 2022 16:04    Titel: Antworten mit Zitat

Man sagt, daß eine Funktion f die Periode T hat, wenn für alle x gilt f(x+T) = f(x). Der Kosinus hat die Periode . Was folgt daraus für

?

_________________
It is just this lack of connection to a concern with truth -- this indifference to how things really are -- that I regard as of the essence of bullshit. -- Harry G. Frankfurt
frage1



Anmeldungsdatum: 20.02.2021
Beiträge: 569
Wohnort: bayern

Beitrag frage1 Verfasst am: 27. Jun 2022 16:06    Titel: Antworten mit Zitat

Dass der cos um 2ka verschoben ist? Und dass man am Ende wieder auf cos(2pi) kommt?
index_razor



Anmeldungsdatum: 14.08.2014
Beiträge: 3259

Beitrag index_razor Verfasst am: 27. Jun 2022 16:12    Titel: Antworten mit Zitat

frage1 hat Folgendes geschrieben:
Dass der cos um 2ka verschoben ist?


Nein. Der Kosinus hat die Periode , d.h. für alle x. Jetzt schau noch mal scharf hin. Was folgt daraus für



Nicht raten, sondern nachdenken. So schwer ist das nicht.

_________________
It is just this lack of connection to a concern with truth -- this indifference to how things really are -- that I regard as of the essence of bullshit. -- Harry G. Frankfurt
frage1



Anmeldungsdatum: 20.02.2021
Beiträge: 569
Wohnort: bayern

Beitrag frage1 Verfasst am: 27. Jun 2022 16:21    Titel: Antworten mit Zitat

Wenn cos (x+2pi)= cos(x), dann muss cos(2ka+2pi)= cos(2ka) sein und in Summe muss gelten: cos(2ka)-cos(2ka)=0

Aber warum ist cos(x+2pi)=cos(x)?? Woher weiß man das? Wieso kann ich 2 pi einfach weglassen bzw. warum ist cos(x+2pi) das gleiche wie cos(x)?
index_razor



Anmeldungsdatum: 14.08.2014
Beiträge: 3259

Beitrag index_razor Verfasst am: 27. Jun 2022 16:40    Titel: Antworten mit Zitat

frage1 hat Folgendes geschrieben:
Wenn cos (x+2pi)= cos(x), dann muss cos(2ka+2pi)= cos(2ka) sein und in Summe muss gelten: cos(2ka)-cos(2ka)=0


Ganz genau. Und a und k spielen dabei keine Rolle.

Zitat:

Aber warum ist cos(x+2pi)=cos(x)?? Woher weiß man das? Wieso kann ich 2 pi einfach weglassen bzw. warum ist cos(x+2pi) das gleiche wie cos(2ka)?


Das weiß man, wenn man sich ein bißchen mit den Eigenschaften von trigonometrischen Funktionen beschäftigt hat. Die Periodizität ist eine ihrer grundlegendsten Eigenschaften.

Mach dir das anhand der Definition am Einheitskreis klar. Das Bogenmaß von entspricht einer vollen Umdrehung. Wenn du also den Winkel in der Abbildung um weiterdrehst, landest du wieder auf demselben Punkt am Einheitskreis. Also ist dort auch der Wert des Kosinus derselbe. Und zwar unabhängig vom Startwinkel .

_________________
It is just this lack of connection to a concern with truth -- this indifference to how things really are -- that I regard as of the essence of bullshit. -- Harry G. Frankfurt
frage1



Anmeldungsdatum: 20.02.2021
Beiträge: 569
Wohnort: bayern

Beitrag frage1 Verfasst am: 27. Jun 2022 18:06    Titel: Antworten mit Zitat

Achso, jetzt versteh' ich's. Es ist quasi egal wo ich beginne. Wenn ich als startwinkel beispielsweise 2ka+2pi habe und diesen um 2pi drehe, dann lande ich wieder am selben Punkt und zwar bei 2ka+2pi und das ist dasselbe wie 2ka. Stimmt das so?
index_razor



Anmeldungsdatum: 14.08.2014
Beiträge: 3259

Beitrag index_razor Verfasst am: 27. Jun 2022 18:29    Titel: Antworten mit Zitat

Der Kosinus hängt nur vom Punkt auf dem Einheitskreis ab. Egal an welchem Punkt du startest, nach einer kompletten Umrundung bist du wieder am selben Punkt mit demselben Kosinus, während sich der Winkel um genau geändert hat. Also muß gelten .

frage1 hat Folgendes geschrieben:
Achso, jetzt versteh' ich's. Es ist quasi egal wo ich beginne. Wenn ich als startwinkel beispielsweise 2ka+2pi habe und diesen um 2pi drehe, dann lande ich wieder am selben Punkt und zwar bei 2ka+2pi und das ist dasselbe wie 2ka. Stimmt das so?


Ja, ich denke jetzt hast du es. Du kannst als Startwinkel natürlich wählen und nach gehen oder du startest bei und gehst nach . Der Kosinus ist an allen diesen Winkeln derselbe, da sie alle zum selben Punkt auf dem Einheitskreis gehören.

_________________
It is just this lack of connection to a concern with truth -- this indifference to how things really are -- that I regard as of the essence of bullshit. -- Harry G. Frankfurt
frage1



Anmeldungsdatum: 20.02.2021
Beiträge: 569
Wohnort: bayern

Beitrag frage1 Verfasst am: 27. Jun 2022 20:25    Titel: Antworten mit Zitat

Verstanden, vielen lieben Dank!!
Und in der Angabe ist ja auch noch gefragt, welche Impulsmesswerte auftreten können. Sind damit die Eigenwerte gemeint? Also in dem Fall die konstanten Vorfaktoren √3/2*√Omega und 1/2*√Omega ?
index_razor



Anmeldungsdatum: 14.08.2014
Beiträge: 3259

Beitrag index_razor Verfasst am: 28. Jun 2022 08:21    Titel: Antworten mit Zitat

frage1 hat Folgendes geschrieben:

Und in der Angabe ist ja auch noch gefragt, welche Impulsmesswerte auftreten können. Sind damit die Eigenwerte gemeint? Also in dem Fall die konstanten Vorfaktoren √3/2*√Omega und 1/2*√Omega ?


Ja, es sind bestimmte Eigenwerte gemeint. Aber die Vorfaktoren sind keine Eigenwerte.

Du hast hier eine Linearkombination von Impulseigenfunktionen gegeben, nämlich



Die beiden sind zwei Eigenfunktionen des Impulsoperators, d.h.



Nach diesen beiden Eigenwerten ist gefragt.

_________________
It is just this lack of connection to a concern with truth -- this indifference to how things really are -- that I regard as of the essence of bullshit. -- Harry G. Frankfurt
frage1



Anmeldungsdatum: 20.02.2021
Beiträge: 569
Wohnort: bayern

Beitrag frage1 Verfasst am: 28. Jun 2022 09:02    Titel: Antworten mit Zitat

Ich muss ja dann die Eigenwerte berechnen. Muss ich dann mit den linerakombinationen rechnen oder nur mit der Wellenfunktion ohne die Linearkombination? Ich versteh´generell nicht was diese Linearkombination aussagen soll


28FE587B-AFDA-4D98-94CE-106B61502556.jpeg
 Beschreibung:

Download
 Dateiname:  28FE587B-AFDA-4D98-94CE-106B61502556.jpeg
 Dateigröße:  37.24 KB
 Heruntergeladen:  78 mal

index_razor



Anmeldungsdatum: 14.08.2014
Beiträge: 3259

Beitrag index_razor Verfasst am: 28. Jun 2022 10:23    Titel: Antworten mit Zitat

Die Linearkombination ist eine Superposition von verschiedenen Eigenfunktionen des Impulsoperators. Diese Superposition ist ein Zustand, in dem verschiedene Impulswerte mit verschiedenen Wahrscheinlichkeiten auftreten können. (Die Wahrscheinlichkeiten sind übrigens die Betragsquadrate der Vorfaktoren in der Linearkombination.) Du mußt nun also zu jeder Eigenfunktion in der Linearkombination den zugehörigen Eigenwert bestimmen.

Die Rechnung aus deinem letzten Beitrag sieht schon fast in Ordnung aus. Leider baust du immer irgendwelche Flüchtigkeitsfehler ein, z.B. verschwinden mitten in der Rechnung die beiden Normierungskonstanten A und B, die ohnehin gleich sind . Aber davon abgesehen, welche Impulswerte können nun auftreten?

_________________
It is just this lack of connection to a concern with truth -- this indifference to how things really are -- that I regard as of the essence of bullshit. -- Harry G. Frankfurt
Neue Frage »
Antworten »
    Foren-Übersicht -> Quantenphysik